Math, asked by vartakshraddha41, 1 day ago

1. As shwon in figure, LK = 6 V2 then 1) MK = ? 2) ML = ? 3) MN
= ?

Attachments:

Answers

Answered by amitnrw
2

Given : LK = 6 √2

To Find :   MK

ML

MN

Solution:

in right angle triangle MLK

Cos 30°  = LK/ MK

=> √3/ 2  = 6√2 / MK

=> MK = 12 √2 / √3

=> MK = 4√3√2

=> MK = 4√6

Tan 30°  = ML/LK

=> 1/√3  = ML / 6√2

=> ML = 6√2 / √3

=> ML = 2√3√2

=> ML = 2√6

in right angle triangle MKN

Sin 45° = MK/MN

=> 1/√2  = 4√3√2 / MN

=> MN = 8√3

Learn More:

if sin@cos@=12/25 then find the sum of all possible value of tan ...

brainly.in/question/6766913

sin theta- cos theta

brainly.in/question/13371875

Answered by riyav1108
0

Step-by-step explanation:

Given : LK = 6 √2

To Find :   MK

ML

MN

Solution:

in right angle triangle MLK

Cos 30°  = LK/ MK

=> √3/ 2  = 6√2 / MK

=> MK = 12 √2 / √3

=> MK = 4√3√2

=> MK = 4√6

Tan 30°  = ML/LK

=> 1/√3  = ML / 6√2

=> ML = 6√2 / √3

=> ML = 2√3√2

=> ML = 2√6

in right angle triangle MKN

Sin 45° = MK/MN

=> 1/√2  = 4√3√2 / MN

=> MN = 8√3

Similar questions